1answer.
Ask question
Login Signup
Ask question
All categories
  • English
  • Mathematics
  • Social Studies
  • Business
  • History
  • Health
  • Geography
  • Biology
  • Physics
  • Chemistry
  • Computers and Technology
  • Arts
  • World Languages
  • Spanish
  • French
  • German
  • Advanced Placement (AP)
  • SAT
  • Medicine
  • Law
  • Engineering
BARSIC [14]
3 years ago
11

Can someone help me PLS

Mathematics
2 answers:
jolli1 [7]3 years ago
8 0

Answer:

Step-by-step explanation:

2^2=1.75^2+3^2-2*3*1.75cos x

4=3.0625+9-10.5 cosx

10.5cos x=9-4+3.0625

10.5cosx=8.0625

cos x= 8.0625/10.5≈0.77

x≈40 degrees

Reptile [31]3 years ago
5 0

Answer:  D) 40 degrees

<u>Step-by-step explanation:</u>

Let  A = θ

then a = 2       b = 3     c = 1.75

Use the Law of Cosines:

a² = b² + c² - 2bc · cos A

2² = 3² + 1.75² - 2(3)(1.75) · cos θ

4 = 9 + 3.0625 - 10.5 cos θ

4 = 12.0625 - 10.5 cos θ

-8.0625 = -10.5 cos θ

\dfrac{-8.0625}{-10.5}=\cos \theta\\\\\\\cos^{-1}\bigg(\dfrac{-8.0625}{-10.5}\bigg)=\theta\\\\\\\large\boxed{39.838=\theta}

You might be interested in
Simplify.<br><br><br><br><br><br><br><br> im stuck on this one need help !
taurus [48]
C is the answer radical 77/22
4 0
3 years ago
Read 2 more answers
Hello, Brainly community!
ioda

Answer:

(B)  \displaystyle \frac{W(3.1) - W(2.9)}{0.2}

General Formulas and Concepts:

<u>Calculus</u>

Limits

Derivatives

  • The definition of a derivative is the slope of the tangent line.

Derivative Notation

Instantaneous Rates

  • Tangent Line: \displaystyle f'(x) = \frac{f(b) - f(a)}{b - a}

Step-by-step explanation:

Since we are trying to find a <em>rate</em> at which W(t) changes, we must find the <em>derivative</em> at <em>t</em> = 3.

We are given 2 close answer choices that would have the same <em>numerical</em> answer but different <em>meanings</em>:

  1. (A)  \displaystyle  \lim_{t \to 3} W(t)
  2. (B)  \displaystyle \frac{W(3.1) - W(2.9)}{0.2}

If we look at answer choice (A), we see that our units would simply just be volume. It would not have the units of a rate of change. Yes, it may be the closest numerically correct answer, but it does not tell us the <em>rate</em> at which the volume would be changing and it is not a derivative.

If we look at answer choice (B), we see that our units would be cm³/s, and that is most certainly a rate of change. Answer choice (B) is also a <em>derivative</em> at <em>t</em> = 3, and a derivative tells us what <em>rate</em> something is changing.

∴ Answer choice (B) will give us the best estimate for the value of the instantaneous rate of change of W(t) when <em>t</em> = 3.

Topic: AP Calculus AB/BC (Calculus I/I + II)

Unit: Differentiation

Book: College Calculus 10e

8 0
2 years ago
mollys jump rope is 6 1/3 feet long. Gals jump rope 4 2/3 feet lo g. How much longer is Molly's jump rope?
Vlad1618 [11]
6 1/3- 4 2/3 =5 4/3- 4 2/3=1 2/3
You can't subtract fractions unless you have the same denominator.
8 0
2 years ago
Read 2 more answers
Please help with these questions!
Colt1911 [192]
1/8 + 6/8 = 7/8
17 11/13 - 9 7/13 = 8 4/13
2/8 + 4/8 = 6/8 = 3/4
8/4 + 1/4 = 9/4 = 2 1/4
9/10 + 3/10 = 12/10 = 1 1/5
2 - 3/5 = 10/5 - 3/5 = 7/5 = 1 2/5
3/4 * 4 = 12/4 = 3
1/4 * 6 = 6/4 = 1 1/2
8 0
3 years ago
A circular pool has a diameter of 40 feet and is surrounded by a wooden deck that has a width of 4 feet and a depth of 6 inches.
Dvinal [7]
40x4x6= 960 square inches
8 0
2 years ago
Other questions:
  • 4% of what number is 5
    7·2 answers
  • Which equation has a constant of proportionality equal to 0.25
    7·1 answer
  • Simplify –7 (–3)x 5 (–6)x
    13·1 answer
  • What does a equal? a/15 = 4/5?
    15·2 answers
  • Please help I will mark brainliest!!! (10 points)
    14·2 answers
  • Can someone please help me with my homework:(
    14·2 answers
  • How do<br> you<br> write 9.59 x 10^2 in a standard form?
    7·1 answer
  • Solve for x in the following equation <br> 2^(8x-8)=4^7x
    15·1 answer
  • On a coordinate plane, which point lies on the x-axis?
    13·1 answer
  • . What is the place value of the 3 in the number 2.703? 3 0.03 0.3 0.003 HERRY IM TIMED I WILL GIVE
    6·1 answer
Add answer
Login
Not registered? Fast signup
Signup
Login Signup
Ask question!